Understanding Limit Questions: Approaches and Solutions | Homework Help

  • Thread starter Thread starter AndreTheGiant
  • Start date Start date
  • Tags Tags
    Limit
Click For Summary
The discussion revolves around evaluating limit questions as they approach (0,0). For the first limit, the user finds that approaching from both the x-axis and y-axis yields -3/0, suggesting the limit does not exist, but seeks confirmation on other methods to analyze it. In the second limit, different approaches from the x-axis and y-axis yield different results (1 and 3), leading to the conclusion that this limit also does not exist. Forum members advise posting fewer problems per thread for better responses and suggest checking limits from different directions for clarity. Overall, the user is encouraged to refine their approach to limit evaluation.
AndreTheGiant
Messages
26
Reaction score
0

Homework Statement


Hello. I have a few questions I wanted to run by just to make sure I did them right.

all the below have lim (x,y) -> (0,0)

1) [PLAIN]http://www4d.wolframalpha.com/Calculate/MSP/MSP188519hd2c481i6c46be00004f512a01afdig9ge?MSPStoreType=image/gif&s=27&w=111&h=40

2) [PLAIN]http://www4d.wolframalpha.com/Calculate/MSP/MSP142419hd2d04f3fccfb70000144ef2c7017g2919?MSPStoreType=image/gif&s=43&w=63&h=43

Homework Equations


The Attempt at a Solution



ok so this is what i did for the following.

1) if i approach from the x-axis and y axis, both the limits give me -3/0 as an answer. If i approach from y = ax, i also get -3/0 as the final answer. So the limit does not exist? What other methods can i use here because I'm not sure if this is right.

2) approaching from the y-axis I get lim y-> 0 of y^2/y^2 = 1
approaching from the x-axis i get lim x-> 0 3x^2/x^2 = 3
since those two aren't equal I can conclude that the limit does not exist.
 
Last edited by a moderator:
Physics news on Phys.org
Andre,

Welcome to PF.

You will get better response if you post only one or two problems per thread. (That's also in the Forum Rules.)

For (1):

You might still want to see if the limit is +∞ or if it's -∞ -- or just plain DOES NOT EXIST. Check that along the y-axis from the positive versus the negative direction.

For (2):

That's correct.

For (3):

Also correct.
 
Thanks for the reply. I edited my post to have 2 questions only. Thanks!
 
Post the other two in another thread
 
Question: A clock's minute hand has length 4 and its hour hand has length 3. What is the distance between the tips at the moment when it is increasing most rapidly?(Putnam Exam Question) Answer: Making assumption that both the hands moves at constant angular velocities, the answer is ## \sqrt{7} .## But don't you think this assumption is somewhat doubtful and wrong?

Similar threads

  • · Replies 13 ·
Replies
13
Views
2K
  • · Replies 5 ·
Replies
5
Views
2K
Replies
24
Views
3K
Replies
2
Views
3K
  • · Replies 10 ·
Replies
10
Views
2K
  • · Replies 5 ·
Replies
5
Views
1K
  • · Replies 2 ·
Replies
2
Views
1K
  • · Replies 3 ·
Replies
3
Views
2K
  • · Replies 3 ·
Replies
3
Views
2K
  • · Replies 2 ·
Replies
2
Views
1K